How Can I Solve This Annoyingly Difficult Integral for Homework?

  • Thread starter Thread starter mwaso
  • Start date Start date
  • Tags Tags
    Integral
Click For Summary
The integral in question is dx/(1-x^2)^(3/2). The initial approach involved recognizing the derivative of inverse sine but struggled with the cube in the expression. A u substitution was attempted but proved ineffective due to complications with the derivative. The discussion highlights the importance of using trigonometric identities, specifically suggesting the substitution x = sin(θ), which simplifies the integral to ∫sec²(θ) dθ. This method ultimately leads to a clearer path for solving the integral.
mwaso
Messages
11
Reaction score
0
Homework Statement

integrate dx/(1-x^2)^(3/2)

The attempt at a solution

I first noticed that the derivative of inverse sin equals (1 x^2)^(-1/2) and that my equation is that derivative cubed. However, I had no idea what to do with the cube, so I dicarded that thought.

Next I tried to do a u substitution. obviously I can't make u equal 1-x^2 because that leaves me with an extra x in my du that won't go away. I was inspired by remembering some powers of trig integrations to try to make du equal the derivative of inverse sin, but that also failed as I had no inverse sin in the original equation to replace with u and last I checked, I can't integrate du^3.

I finally checked my tables of integrals and found where the integral of du/(a^2-u^2)^(3/2) equals u/a^2(a^2-u^2). This seems to give me my answer, but I can't figure out how to derive that formula.
 
Physics news on Phys.org
From I=\int\frac{dx}{(1-x^2)^{\frac{3}{2}}} apply the substitution x=\sin\theta. Hint: you should get I=\int\sec ^2 \theta\, d\theta.
 
You probably substituted wrongly to get du^3
 
Question: A clock's minute hand has length 4 and its hour hand has length 3. What is the distance between the tips at the moment when it is increasing most rapidly?(Putnam Exam Question) Answer: Making assumption that both the hands moves at constant angular velocities, the answer is ## \sqrt{7} .## But don't you think this assumption is somewhat doubtful and wrong?

Similar threads

  • · Replies 1 ·
Replies
1
Views
2K
  • · Replies 15 ·
Replies
15
Views
2K
  • · Replies 8 ·
Replies
8
Views
2K
  • · Replies 22 ·
Replies
22
Views
3K
  • · Replies 3 ·
Replies
3
Views
2K
  • · Replies 1 ·
Replies
1
Views
2K
  • · Replies 5 ·
Replies
5
Views
2K
  • · Replies 3 ·
Replies
3
Views
2K
  • · Replies 12 ·
Replies
12
Views
3K
  • · Replies 27 ·
Replies
27
Views
3K